Fundamental Forces: finding velocity using different fields and forces

In summary, the conversation is about solving a problem involving a particle with known acceleration and net force, and finding the ratio of two unknown velocity vectors. The given hints include using the triple vector product and the fact that the velocity is perpendicular to the magnetic field. The conversation also includes discussing the mistake made in calculating a cross product and the need to write down the determinant and determine the components.
  • #1
JoeyBob
256
29
Homework Statement
See attached
Relevant Equations
F=qE, F=qv cross product B
So I know the acceleration is 0, so the net force is 0.

QE=1.6E-3*9.9E3 k hat = 15.84 k hat (thats one force)

qv x B = q(v i hat + v j hat) x (0.51 i hat + 0.10 j hat)

=q(0.10 v k hat- 0.51 v k hat)
=q(-0.41 v k hat)
=-0.000656 v k hat

Now solving for velocity,

0.000656v=15.84, v=24146.34146

The question says I need to find the ratio, and I know vy is .41 times higher, so i multiply by this to get 9890.

This answer is wrong, it should be -3665.31.
 

Attachments

  • Question.PNG
    Question.PNG
    16.5 KB · Views: 105
Physics news on Phys.org
  • #2
JoeyBob said:
v i hat + v j hat
I don't understand what you mean by this. Elsewhere, you interpret v as the scalar speed, but above you seem to mean ##v_i\hat i+v_j\hat j##.
Btw, there is a neater way than the given hint.
Given equations ##\vec X\times\vec Y=\vec Z## and ##\vec X.\vec Y=A## you can take the triple vector product ##\vec Y\times\vec Z=\vec Y\times(\vec X\times\vec Y)=(\vec Y.\vec Y)\vec X-(\vec Y.\vec X)\vec Y=Y^2\vec X-A\vec Y##.
Whence ##\vec X= \frac{\vec Y\times\vec Z+A\vec Y}{Y^2}##.
 
Last edited:
  • #3
haruspex said:
I don't understand what you mean by this. Elsewhere, you interpret v as the scalar speed, but above you seem to mean ##v_i\hat i+v_j\hat j##.
Btw, there is a neater way than the given hint.
Given equations ##\vec X\times\vec Y=\vec Z## and ##\vec X.\vec Y=A## you can take the triple vector product ##\vec Y\times\vec Z=\vec Y\times(\vec X\times\vec Y)=(\vec Y.\vec Y)\vec X-(\vec Y.\vec X)\vec Y=Y^2\vec X-A\vec Y##.
Whence ##\vec X= \frac{\vec Y\times\vec Z+A\vec Y}{Y^2}##.

How are you suppose to use the hint?
 
  • #4
Perhaps it would be conceptually easier for you to write two equations, one saying that the net force on the particle is zero and another saying that the velocity is perpendicular to the magnetic field. You will have two equations and two unknowns, ##v_x## and ##v_y##.

Don't forget that the net force is the sum of an electric force and a magnetic force.
 
  • Like
Likes Delta2
  • #5
kuruman said:
Perhaps it would be conceptually easier for you to write two equations, one saying that the net force on the particle is zero and another saying that the velocity is perpendicular to the magnetic field. You will have two equations and two unknowns, ##v_x## and ##v_y##.

Don't forget that the net force is the sum of an electric force and a magnetic force.

So one force would be F=qE. I think its the magnetic field force that is confusing me.

Would it just be F=q(##v_x##+##v_y##) x B?

Then these forces added together are 0?

Thats what I did when I tried to solve it, but I got stuck because I have two unknown variables now (the two different velocity vectors).
 
  • #6
JoeyBob said:
qv x B = q(v i hat + v j hat) x (0.51 i hat + 0.10 j hat)

=q(0.10 v k hat- 0.51 v k hat)
=q(-0.41 v k hat)
=-0.000656 v k hat

You made a mistake in this calculation of cross product. Maybe you can write down the determinant explicitly and try to figure out what are the components.

JoeyBob said:
Thats what I did when I tried to solve it, but I got stuck because I have two unknown variables now (the two different velocity vectors).

There is another hint in your question that you didn't use - the velocity is perpendicular to magnetic field. As mentioned above, you will have two variables and two equations.
 
  • #7
JoeyBob said:
Would it just be F=q(##v_x##+##v_y##) x B?
It would not. Please (re)view how to write the cross product in terms of its components here to see what it would be.
JoeyBob said:
Thats what I did when I tried to solve it, but I got stuck because I have two unknown variables now (the two different velocity vectors).
I know one velocity vector, the one associated with the particle mentioned in the problem which is ##\vec v=v_x~\hat x+v_y~\hat y.## What velocity vector is the other one and what entity has it?
 
  • #8
JoeyBob said:
How are you suppose to use the hint?
My vector triple product hint or the one they gave?
 
  • #9
pai535 said:
You made a mistake in this calculation of cross product. Maybe you can write down the determinant explicitly and try to figure out what are the components.
There is another hint in your question that you didn't use - the velocity is perpendicular to magnetic field. As mentioned above, you will have two variables and two equations.

What equation is that?
 
  • #10
JoeyBob said:
What equation is that?
##\vec B\cdot \vec v=0.##
 
  • Like
Likes pai535
  • #11
kuruman said:
##\vec B\cdot \vec v=0.##
Wouldn't that just mean the velocity is 0 then?
 
  • #12
No. It's another way of saying that the two vectors are perpendicular. I suggest tht you review dot products and cross products.
 
  • #13
kuruman said:
No. It's another way of saying that the two vectors are perpendicular. I suggest tht you review dot products and cross product.

So then the velocity is either -1.96 i hat + 10 j hat or 1.96 i hat - 10 j hat if its perpendicular. But this doesn't really help me.
 
  • #14
JoeyBob said:
So then the velocity is either -1.96 i hat + 10 j hat or 1.96 i hat - 10 j hat if its perpendicular. But this doesn't really help me.
How did you arrive at these numbers? Are these the only choices that make the magnetic field and velocity perpendicular? Remember that the particle has to go in a straight line. As I mentioned before, the net force on the particle must be zero because we are told that it is moving in a straight line. See post #4.
 
  • #15
kuruman said:
How did you arrive at these numbers?

Because with the dot product they give 1-1, which is 0. I don't see why it wouldn't be a straight line, it would go in the direction of the vectors.
 
  • #16
Which direction is this and which vectors are you talking about? There is an electric field along the z-axis and the magnetic field in the xy plane. The particle's velocity is perpendicular to both the electric and magnetic field. Therefore it is not movinfg along either field.

Having the velocity perpendicular to the field is not enough to ensure that the particle is moving in a straight line. That's because the magnetic force on the particle is velocity-dependent. The velocity has to be just right so that the sum of the electric and magnetic forces on the particle is zero.
 
  • #17
kuruman said:
Which direction is this and which vectors are you talking about? There is an electric field along the z-axis and the magnetic field in the xy plane. The particle's velocity is perpendicular to both the electric and magnetic field. Therefore it is not movinfg along either field.

Having the velocity perpendicular to the field is not enough to ensure that the particle is moving in a straight line. That's because the magnetic force on the particle is velocity-dependent. The velocity has to be just right so that the sum of the electric and magnetic forces on the particle is zero.

I know B=0.51 i hat + 0.10 j hat. I need to find the ratio of vx to vy by making the dot product =0.

(0.51 i hat + 0.10 j hat) * (1.96 i hat -10 j hat) =0. (0.51 i hat + 0.10 j hat) * (-1.96 i hat + 10 j hat) is also 0. I assume this means that the vx has a ratio of 0.196 to vy.
 
  • #18
Sure, but that is not enough to determine vx and vy separately. It is also true that
(0.51 i hat + 0.10 j hat) * (2*1.96 i hat -2*10 j hat) = 0
(0.51 i hat + 0.10 j hat) * (23.4*1.96 i hat -23.4*10 j hat) = 0
(0.51 i hat + 0.10 j hat) * (1065*1.96 i hat -1065*10 j hat) = 0
...
And so on. You get the picture? Like I said, of all these infinitely many choices of velocities perpendicular to the magnetic field, only one is correct and will result in the particle following a straight line. The equation saying that the net force is zero will give that correct choice.
 
  • Like
Likes JoeyBob
  • #19
kuruman said:
Sure, but that is not enough to determine vx and vy separately. It is also true that
(0.51 i hat + 0.10 j hat) * (2*1.96 i hat -2*10 j hat) = 0
(0.51 i hat + 0.10 j hat) * (23.4*1.96 i hat -23.4*10 j hat) = 0
(0.51 i hat + 0.10 j hat) * (1065*1.96 i hat -1065*10 j hat) = 0
...
And so on. You get the picture? Like I said, of all these infinitely many choices of velocities perpendicular to the magnetic field, only one is correct and will result in the particle following a straight line. The equation saying that the net force is zero will give that correct choice.
But the ratio is always the same, no?

I understand that 0=qE+qv x B, but idk how to use the ratios.
 
  • #20
Write the equation ##q\vec E+q\vec v\times \vec B=0## correctly in terms of ##v_x## and ##v_y##. I gave you a link in post #7 if you don't know how to do that. That is the second equation, the first one being ##B_x v_x+B_y v_y=0.##
 
  • Like
Likes JoeyBob and pai535
  • #21
JoeyBob said:
But the ratio is always the same, no?

I understand that 0=qE+qv x B, but idk how to use the ratios.

Using the ratio, you can express ##v_y## in terms of ##v_x## (also ##v_x## in terms of ##v_y##). Then you reduce the problem to a problem with only one unknown variable.
 
  • Like
Likes JoeyBob
  • #22
pai535 said:
Using the ratio, you can express ##v_y## in terms of ##v_x## (also ##v_x## in terms of ##v_y##). Then you reduce the problem to a problem with only one unknown variable.

okay here's what I did.

Knowing B x (vx + vy)=0, I found the ratio of vx to vy was -0.196

Now I also know 0=qE+qv x B

-E=v x B

-E=(vx i hat -vx/0.196 j hat) x (0.51 i hat + 0.10 j hat)

-E=0.10vx k hat+2.602vx k hat

-E/(0.1+2.602)=vx=-3664

This is one less than the right answer so I am assuming i made some rounding error.

Thanks for the help everyone! Took me a while but I got it...
 
  • Like
Likes kuruman
  • #23
JoeyBob said:
B x (vx + vy)=0,
For future reference, use the symbol "x" to denote cross product only. Here the context is dot product, so use a dot. Also, use unit vectors where applicable. (vx + vy) looks like you are adding the two components of a vector as scalars, not as vectors n orthogonal directions. This is important if you seek help and need to be understood because although you know what you mean, the reader may not.

I would strongly recommend that you invest some time to learn how to use LaTeX to write equations. It's a skill that will serve you well if you follow a technical career. PF provides a good LaTeX starter guide if you click "LaTeX Guide" on this screen, bottom left, just above "Attach files".
 
  • Like
Likes Steve4Physics, pai535 and JoeyBob
  • #24
kuruman said:
For future reference, use the symbol "x" to denote cross product only. Here the context is dot product, so use a dot. Also, use unit vectors where applicable. (vx + vy) looks like you are adding the two components of a vector as scalars, not as vectors n orthogonal directions. This is important if you seek help and need to be understood because although you know what you mean, the reader may not.

I would strongly recommend that you invest some time to learn how to use LaTeX to write equations. It's a skill that will serve you well if you follow a technical career. PF provides a good LaTeX starter guide if you click "LaTeX Guide" on this screen, bottom left, just above "Attach files".
Im doing a chemistry focused degree, so idk how beneficial learning LaTeX would be in the furute. Have to take first year physics then I am done with it.

Personally physics isn't that bad, but its not something I would want to pursue a career in.
 
  • #25
JoeyBob said:
Im doing a chemistry focused degree, so idk how beneficial learning LaTeX would be in the furute. Have to take first year physics then I am done with it.

Personally physics isn't that bad, but its not something I would want to pursue a career in.

Actually LaTeX is everywhere in science, not just physics. If you want to write and publish scientific articles (and even books), you'd better learn some LaTeX. Probably there will also be formulas and calculations in your future study (although I don't know much about chemistry). Using LaTeX can make them much easier to read.
 
  • #26
JoeyBob said:
Took me a while but I
Here's my method:
##\vec E+\vec v\times\vec B=0##
##\vec v.\vec B=0##
Taking the cross product of the first equation with ##\vec B##:
##0=\vec B\times\vec E+\vec B\times(\vec v\times\vec B)=\vec B\times\vec E+(\vec B.\vec B)\vec v-(\vec B.\vec v)\vec B=\vec B\times\vec E+(\vec B.\vec B)\vec v##
Whence ##\vec v=-\frac{\vec B\times\vec E}{\vec B.\vec B}##
 
  • #27
haruspex said:
Here's my method:
##\vec E+\vec v\times\vec B=0##
##\vec v.\vec B=0##
Taking the cross product of the first equation with ##\vec B##:
##0=\vec B\times\vec E+\vec B\times(\vec v\times\vec B)=\vec B\times\vec E+(\vec B.\vec B)\vec v-(\vec B.\vec v)\vec B=\vec B\times\vec E+(\vec B.\vec B)\vec v##
Whence ##\vec v=-\frac{\vec B\times\vec E}{\vec B.\vec B}##
Alternatively, the set up can be recognized as a velocity selector in which case the speed is equal to the ratio of the field magnitudes and the direction perpendicular to both crossed fields such that the electric and magnetic forces are antiparallel. Then, $$v=\frac{E}{B}~;~~\hat v=\frac{\vec E\times \vec B}{EB}~~\Rightarrow~\vec v=v~\hat v=\frac{\vec E\times \vec B}{B^2}.$$Note that the three participating unit vectors form a right-handed coordinate system. The result is independent of Cartesian axes and of the sign of the moving charge.
 
Last edited:
  • #28
kuruman said:
Alternatively, the set up can be recognized as a velocity selector in which case the speed is equal to the ratio of the field magnitudes and the direction perpendicular to both crossed fields such that the electric and magnetic forces are antiparallel. Then, $$v=\frac{E}{B}~;~~\hat v=\frac{\vec E\times \vec B}{EB}~~\Rightarrow~\vec v=v~\hat v=\frac{\vec E\times \vec B}{B^2}.$$Note that the three participating unit vectors form a right-handed coordinate system. The result is independent of Cartesian axes and of the sign of the moving charge.
Above my level I think.
 
  • #29
JoeyBob said:
Above my level I think.
Just think of an apparatus that if you send in a beam of charged particles, only the ones with a specific speed can travel through. Then you can sort out all the particles with the speed you want (by adjusting E and M fields).
 

1. What are the four fundamental forces in physics?

The four fundamental forces in physics are gravity, electromagnetism, strong nuclear force, and weak nuclear force. These forces govern the interactions between particles and objects in the universe.

2. How do fundamental forces affect the velocity of an object?

The fundamental forces can affect the velocity of an object by either accelerating or decelerating it. For example, gravity can accelerate an object towards the ground, while friction can decelerate an object's motion.

3. Can different fields and forces be used to find an object's velocity?

Yes, different fields and forces can be used to find an object's velocity. For example, the force of gravity and the strength of an electric field can be used to calculate the velocity of a falling object.

4. How do you calculate the velocity using different fields and forces?

To calculate the velocity using different fields and forces, you will need to use Newton's second law of motion, which states that the net force acting on an object is equal to its mass multiplied by its acceleration. By rearranging this equation, you can solve for the velocity of the object.

5. What are some real-life applications of using fundamental forces to find velocity?

Some real-life applications of using fundamental forces to find velocity include predicting the trajectory of a projectile, calculating the speed of a roller coaster, and determining the acceleration of a car on a hill. These calculations are crucial in fields such as engineering, physics, and astronomy.

Similar threads

  • Introductory Physics Homework Help
Replies
12
Views
211
  • Introductory Physics Homework Help
Replies
11
Views
232
  • Introductory Physics Homework Help
Replies
6
Views
165
  • Introductory Physics Homework Help
Replies
2
Views
194
  • Introductory Physics Homework Help
Replies
25
Views
285
  • Introductory Physics Homework Help
Replies
2
Views
663
  • Introductory Physics Homework Help
Replies
2
Views
785
  • Introductory Physics Homework Help
Replies
3
Views
211
  • Introductory Physics Homework Help
Replies
17
Views
1K
  • Introductory Physics Homework Help
Replies
4
Views
842
Back
Top